Difference between revisions of "1950 AHSME Problems/Problem 23"

(Problem)
(Problem)
Line 1: Line 1:
 
==Problem==
 
==Problem==
  
A man buys a house for &#036;10,000 dollars and rents it. He puts <math>12\frac{1}{2}\%</math> of each month's rent aside for repairs and upkeep; pays  &#036;325 a year taxes and realizes <math>5\frac{1}{2}\%</math> on his investment. The monthly rent (in dollars) is:
+
A man buys a house for &#036;10,000 and rents it. He puts <math>12\frac{1}{2}\%</math> of each month's rent aside for repairs and upkeep; pays  &#036;325 a year taxes and realizes <math>5\frac{1}{2}\%</math> on his investment. The monthly rent (in dollars) is:
  
 
<math> \textbf{(A)}\ \64.82\qquad\textbf{(B)}\ \83.33\qquad\textbf{(C)}\ \72.08\qquad\textbf{(D)}\ \45.83\qquad\textbf{(E)}\ \177.08 </math>
 
<math> \textbf{(A)}\ \64.82\qquad\textbf{(B)}\ \83.33\qquad\textbf{(C)}\ \72.08\qquad\textbf{(D)}\ \45.83\qquad\textbf{(E)}\ \177.08 </math>

Revision as of 17:34, 22 November 2011

Problem

A man buys a house for $10,000 and rents it. He puts $12\frac{1}{2}\%$ of each month's rent aside for repairs and upkeep; pays $325 a year taxes and realizes $5\frac{1}{2}\%$ on his investment. The monthly rent (in dollars) is:

$\textbf{(A)}\ \64.82\qquad\textbf{(B)}\ \83.33\qquad\textbf{(C)}\ \72.08\qquad\textbf{(D)}\ \45.83\qquad\textbf{(E)}\ \177.08$ (Error compiling LaTeX. Unknown error_msg)